Difference between revisions of "2017 AMC 10B Problems/Problem 9"

m (Solution)
(Solution)
Line 7: Line 7:
 
There are two ways that the contestant can win.
 
There are two ways that the contestant can win.
  
Case 1: They guess all three right. This can only happen <math>1/3 * 1/3 * 1/3 = 1/27</math> of the time.
+
Case 1: They guess all three right. This can only happen <math>\frac1/{3} * \frac1/{3} * \frac1/{3} = \frac1/{27}</math> of the time.
  
Case 2: They guess only two right. We pick one of the questions to get wrong, <math>3</math>, and this can happen <math>1/3 * 1/3 * 2/3</math> of the time. Thus, <math>2/27 * 3</math> = <math>6/27</math>.
+
Case 2: They guess only two right. We pick one of the questions to get wrong, <math>3</math>, and this can happen <math>\frac1/{3} * \frac1/{3} * \frac2/{3}</math> of the time. Thus, <math>\frac2/{27} * 3</math> = <math>\frac6/{27}</math>.
  
So, in total the two cases combined equals <math>1/27 + 6/27</math> = <math>\boxed{\textbf{(D)}\ 7/27}</math>.
+
So, in total the two cases combined equals <math>\frac1/{27} + \frac6/{27}</math> = <math>\boxed{\textbf{(D)}\ \frac7/{27]}</math>.
  
 
==See Also==
 
==See Also==
 
{{AMC10 box|year=2017|ab=B|num-b=8|num-a=10}}
 
{{AMC10 box|year=2017|ab=B|num-b=8|num-a=10}}
 
{{MAA Notice}}
 
{{MAA Notice}}

Revision as of 14:40, 16 February 2017

Problem

A radio program has a quiz consisting of $3$ multiple-choice questions, each with $3$ choices. A contestant wins if he or she gets $2$ or more of the questions right. The contestant answers randomly to each question. What is the probability of winning?

$\textbf{(A)}\ \frac{1}{27}\qquad\textbf{(B)}\ \frac{1}{9}\qquad\textbf{(C)}\ \frac{2}{9}\qquad\textbf{(D)}\ \frac{7}{27}\qquad\textbf{(E)}\ \frac{1}{2}$

Solution

There are two ways that the contestant can win.

Case 1: They guess all three right. This can only happen $\frac1/{3} * \frac1/{3} * \frac1/{3} = \frac1/{27}$ of the time.

Case 2: They guess only two right. We pick one of the questions to get wrong, $3$, and this can happen $\frac1/{3} * \frac1/{3} * \frac2/{3}$ of the time. Thus, $\frac2/{27} * 3$ = $\frac6/{27}$.

So, in total the two cases combined equals $\frac1/{27} + \frac6/{27}$ = $\boxed{\textbf{(D)}\ \frac7/{27]}$ (Error compiling LaTeX. Unknown error_msg).

See Also

2017 AMC 10B (ProblemsAnswer KeyResources)
Preceded by
Problem 8
Followed by
Problem 10
1 2 3 4 5 6 7 8 9 10 11 12 13 14 15 16 17 18 19 20 21 22 23 24 25
All AMC 10 Problems and Solutions

The problems on this page are copyrighted by the Mathematical Association of America's American Mathematics Competitions. AMC logo.png